The range, as x tends to negative infinity, the function will tend to zero. And as x tends to infinity, h(x) also tends to infinity.
So the range exists R: (0, ∞).
What is meant by exponential function?A mathematical function with the following formula is an exponential function: f (x) = \($$a^{x}\). where x is a variable and an is a fixed amount known as the function's base. The transcendental number e, or roughly 2.71828, is the most frequently encountered exponential-function base.
Let the exponential function be
\($h(x)=125^x\)
The base, 125 , exists larger than 1 , which means that we contain an exponential growth.
So, as x increases, also does h(x).
As x decreases, also does h(x).
Now, the domain of any exponential equation exists the set of all real numbers.
For the range, as x tends to negative infinity, the function will tend to zero. And as x tends to infinity, h(x) also tends to infinity.
So the range exists R: (0, ∞).
To learn more about exponential equations refer to:
https://brainly.com/question/2456547
#SPJ1
What is the smallest positive number that is prime and 10 less than a perfect square?
Answer:
The problem states that the answer cannot be a perfect square or have prime factors less than $50$. Therefore, the answer will be the product of at least two different primes greater than $50$. The two smallest primes greater than $50$ are $53$ and $59$. Multiplying these two primes, we obtain the number $3127$, which is also the smallest number on the list of answer choices. So we are done, and the answer is $\boxed{\textbf{(A)}\ 3127}$.
Step-by-step explanation:
Answer:
71
Step-by-step explanation:
I just did the AOPS question, you can see the attachment down below.
Hope this helped! :)
Enter
30
to
70 as a fully simplified ratio.
Answer:
3:7
Step-by-step explanation:
To simplify the ratio 30:70, we find the greatest common divisor of 30 and 70, and then we divide 30 and 70 by the greatest common divisor. The greatest common divisor that you can use to simplify 30:70 is 10. This means the answer to ratio 30:70 simplified is: 3:7.
Please mark me brainliest!
. as the statistical consultant to ahmadi, what would you advise them? use a .05 level of significance.
My advice to Ahmadi would be to approach their statistical analysis with care and to consider all aspects of their data and results, not just the significance level. By doing so, they can ensure that their findings are valid, reliable, and meaningful.
As the statistical consultant to Ahmadi, my advice would be to proceed with caution and carefully analyze their data before making any conclusions. The use of a significance level of .05 is a common practice in statistical analysis, but it should not be used as the sole criterion for decision-making.
To begin with, Ahmadi should ensure that their data is reliable and accurate. They should review their data collection methods and procedures to ensure that they are free from bias and error. They should also consider the sample size and make sure that it is large enough to provide a representative sample of their population.
Once they have established the validity of their data, Ahmadi should then conduct a thorough statistical analysis. They should choose appropriate statistical tests based on the nature of their data and research question. They should also be mindful of any assumptions that underlie their tests and make sure that those assumptions are met.
When interpreting their results, Ahmadi should not rely solely on the p-value or significance level. They should also consider the effect size, which provides a measure of the magnitude of the effect they are studying. They should also consider the practical significance of their results and whether they have any real-world implications.
Finally, Ahmadi should be transparent about their statistical methods and results. They should clearly report their methods and results in their publications and presentations so that others can evaluate and replicate their findings.
In summary, my advice to Ahmadi would be to approach their statistical analysis with care and to consider all aspects of their data and results, not just the significance level. By doing so, they can ensure that their findings are valid, reliable, and meaningful.
for more questions on statistical
https://brainly.com/question/12150562
#SPJ11
1. Which of the following is equivalent to 5a³ + 4a³??
O (5+4) a³+3
O (5+4) a³
(5.4) a³+3
O (5.4) a³
ہے
The equivalent to 5a³ + 4a³ is (5+4) a³, according to the question.
What do you mean by Equivalent ?
Equivalent refers to the mathematical notion that distinct terms and expressions having a similar value are treated equally.
According to the given question,
We have the given options are:
O (5+4) a³+3
O (5+4) a³
O (5.4) a³+3
O (5.4) a³
Now, we will find the equivalent,
5a³ + 4a³
Then, we will taking common factor from the given equation:
5a³ + 4a³
= a³(5+4)
So that is same as (5+4)a³.
Therefore, the equivalent to 5a³ + 4a³ is (5+4) a³.
To learn more about equivalent, visit:
https://brainly.com/question/14672772?referrer=searchResults
#SPJ1
WORTH 15 POINTS!!!
Ok so people have told me before that you can’t post links here, but I’m pretty sure you can in the chat area. Please if you can send link of a good water fall and/or mountain painting. (And yes, i know there is gogle and you tube, I just can’t find anything good.)
Answer:
Hi, don't worry, I am not one of those persons who want to answer just because of points. Please look in the chat. Brainliest please...
Step-by-step explanation:
A student can ride his bicycle at an average speed of 6 mph. If he cycles for 4 hours, how far can he travel Sedit 15 miles
A student can ride his bicycle at an average speed of 6 mph. If he cycles for 4 hours, how far can he travel 24 miles.
The overall distance the object covers in a given amount of time is its average speed. A scalar value represents the average speed. It has no direction and is indicated by the magnitude.
We can use the formula:
\(Distance = rate * time\)
To solve this problem.
The rate is given as 6 miles per hour, and the time is given as 4 hours.
So, we have:
distance = 6 * 4
Simplifying, we get:
distance = 24 miles
So, the student can travel 24 miles if he cycles for 4 hours at an average speed of 6 mph.
To learn more about average speed visit;
https://brainly.com/question/12322912
#SPJ4
Find the expected value of the winnings
from a game that has the following
payout probability distribution:
Payout ($) 0 5
8
10
15
Probability 0.5 0.2 0.15 0.1 0.05
Expected Value = [?]
Multiply each payout by the corresponding probability and take the total:
E[X] = 0×0.5 + 5×0.2 + 8×0.15 + 10×0.1 + 15×0.05 = 3.95
The expected value = 3.92
What is expected value ?"It describes the average of a discrete set of variables based on their associated probabilities."
Formula of expected value:\(E(x)=\Sigma[ xP(x)]\)
Multiply each value of the random variable by its probability and add the products.
For given question,
We have been given a payout probability distribution.
We need to find the expected value of the winnings.
First we multiply each value of the random variable by its probability .
0 × 0.5 = 0
5 × 0.2 = 1
8 × 0.15 = 1.2
10 × 0.1 = 1
15 × 0.05 = 0.75
Now, we find the sum of above products.
0 + 1 + 1.2 + 1 + 0.75 = 3.95
By using the formula of expected value,
\(\Rightarrow E(x)=\Sigma[ xP(x)]\\\\\Rightarrow E(x)=3.92\)
Therefore, the expected value = 3.92
Learn more about the expected value here:
https://brainly.com/question/18523098
#SPJ3
Help PLEASEE!!!!!!!!!!
I think it’s the third one. Hope that helps!
Answer:
the answers are B or C x>4/19
question and options are in the picture!
you start at (1,-4). you move left 4units and right 6 units. where do you end
Your final position is at (3, -4) if you move left 4 units and right 6 units after starting at (1, -4).
The initial point is given as (1, -4). The point moves to the left 4 units and right 6 units. Let us assume the starting point is point A. Let B be the point obtained after moving 4 units to the left and C be the point obtained after moving 6 units to the right.
Then,
A = (1, -4),
B = (1-4, -4) = (-3, -4)
C = (-3+6, -4) = (3, -4)
Thus, the final point is (3,-4).
To know more about final position, refer here:
https://brainly.com/question/28569699
#SPJ11
how many x-intercepts does the polynomial function y=(x-2)(x+3)^2 have?
Answer:
2
Step-by-step explanation:
(x-2)(x+3)(x+3)
(x² + x - 6)(x+3)
x³ + 4x² - 3x - 18
we can see here it is cubic but 2 of the roots are the same
At D-Luxe Detailing, teams of employees carefully wash and detail customers' cars. The
function f(x) gives the total number of hours they spend servicing x cars.
What does f(5) < 5 tell you?
In 5 hours, the employees service fewer than 5 cars.
ns
The employees service 5 cars in under 5 hours.
At D-Luxe Detailing, teams of employees, the expression, f(5) < 5 told us that employees service five cars in less than five hours . So, option(b) is right answer.
What is function ?A function is defined as a kind of rule that gives us one output for an input . If we put something for x, we get one output valur for f(x) . We would say that f(x) is a function of x since x is the input value.
We have , At D-Luxe Detailing has a team of employees who carefully wash and detail customers' cars. The function , f(x) represents the total hours which employees spend to wash x cars i.e. f(x) denotes time in hours. Now, the expression is f(5) < 5, here x= 5 that is number of car washed by employees is equals to five. but f(5) < 5 that value of f(x) is less than 5 when we input x = 5 . So, this expression told us that five cars are washed by employees in less than 5 hours.
To learn more about function, refer:
https://brainly.com/question/11624077
#SPJ4
Complete question:
At D-Luxe Detailing, teams of employees carefully wash and detail customers' cars. The
function f(x) gives the total number of hours they spend servicing x cars.
What does f(5) < 5 tell you?
a)In 5 hours, the employees service less than 5 cars.
b) The employees service 5 cars in fewer than five hours.
c)The employees service 5 cars in under 5 hours.
Kelly invested $5,000 in a cd at charter bank. the annual rate is 4.9% compounded monthly. how much is her investment at the end of 8 years? round to the nearest hundredth.
The investment of the kelly at the end of 8 years is $7393.78.
What will be the investment of kelly at the end of 8 years?It is given that
The principle P= $5000
rate of interest R=4.9%
The rate is compounded monthly so
\(CI=P(1+\dfrac{R}{12}^{12t} )\)
By putting the value
\(CI= 5000\times (1+\dfrac{0.049}{12}^{12\times8} )\)
\(CI=7393.78\)
Thus the investment of the kelly at the end of 8 years is $7393.78.
To know more about Compound interest follow
https://brainly.com/question/24924853
how do you solve 83 = y/5?
Answer: 415
83 = \(\frac{y}{5}\)
83(5) = y
415 = y
Answer:
y = 415
Step-by-step explanation:
83 = y/5
83 x 5 = y
415 = y
Need Help here Please!
Answer:
Step-by-step explanation:
To solve the given equation \(\sf x - y = 4 \\\), we can perform the following calculations:
a) To find the value of \(\sf 3(x - y) \\\):
\(\sf 3(x - y) = 3 \cdot 4 = 12 \\\)
b) To find the value of \(\sf 6x - 6y \\\):
\(\sf 6x - 6y = 6(x - y) = 6 \cdot 4 = 24 \\\)
c) To find the value of \(\sf y - x \\\):
\(\sf y - x = - (x - y) = -4 \\\)
Therefore:
a) The value of \(\sf 3(x - y) \\\) is 12.
b) The value of \(\sf 6x - 6y \\\) is 24.
c) The value of \(\sf y - x \\\) is -4.
\(\huge{\mathfrak{\colorbox{black}{\textcolor{lime}{I\:hope\:this\:helps\:!\:\:}}}}\)
♥️ \(\large{\underline{\textcolor{red}{\mathcal{SUMIT\:\:ROY\:\:(:\:\:}}}}\)
Q1: Using MATLAB instruction: \[ z 1=[2+5 i 3+7 i ; 6+13 i 9+11 i], z 2=\left[\begin{array}{lll} 7+2 i & 6+8 i ; 4+4 s q r t(3) i & 6+s q r t(7) i \end{array}\right] \] i. Find z1z2 and display the re
Here is the answer to your question.Q1: Using MATLAB instruction:\(\[ z_1=[2+5 i 3+7 i ; 6+13 i 9+11 i], z_2=\left[\begin{array}{lll} 7+2 i & 6+8 i ; 4+4 s q r t(3) i & 6+s q r t(7) i \end{array}\right] \] i.\) Find z1z2 and display the result in rectangular form.
Since the sizes of z1 and z2 are compatible, we can multiply them. The MATLAB code for multiplying z1 and z2 is shown below:>>z1
=\([2+5i 3+7i; 6+13i 9+11i]; > > z2=[7+2i 6+8i; 4+4*sqrt(3)*i 6+sqrt(7)*i]; > > z1z2=z1*z2 The result of z1z2 is:z1z2\)
= -39.0000 + 189.0000i -50.0000 - 97.0000i -152.0000 - 50.0000i -42.0000 +154.0000iTo represent the result in rectangular form, we need to use the real() and imag() functions to get the real and imaginary parts of the product. .
Then, we can combine these parts using the complex() function to get the result in rectangular form. The MATLAB code for this is shown below:>>rectangular_result
= complex(real(z1z2), imag(z1z2))
= -39.0000 + 189.0000i -50.0000 - 97.0000i -152.0000 - 50.0000i -42.0000 +154.0000i
To know more about imaginary visit:
https://brainly.com/question/197818
#SPJ11
when slicing a pizza a chef tries to make the angle of each slice as shown by angle BAC congruent to the angles of all of the other slices. if half a pizza is cut into slices with congruent angles of 30 degrees each, how many slices of pizza will there be
Answer:
12
Step-by-step explanation:
The whole pizza comprises 360°
If each piece comprises 30° of the circle, there will be ;
360 / 30 = 12 pieces
Which quantity may be calculated directly using Newton’s second law of motion?
6. Find d.
Please help
Answer:
Step-by-step explanation:
The first thing we are going to do is to fill in the other angles that we need to solve this problem. You could find ALL of them but all of them isn't necessary. So looking at the obtuse angle next to the 35 degree angle...we know that those are supplementary so 180 - 35 = the obtuse angle in the small triangle. 180 - 35 = 145. Within the smaller triangle we have now the 145 and the 10, and since, by the Triangle Angle-Sum Theorem all the angles have to add up to equal 180, then 180 - (10 + 145) = the 3rd angle, so the third angle is 180 - 155 = 25. Now let's get to the problem. If I were you, I'd draw that out like I did to keep track of these angles cuz I'm going to name them by their degree. In order to find d, we need to first find the distance between d and the right angle. We'll call that x. The reference angle is 35, the side opposite that angle is 12 and the side we are looking for, x, is adjacent to that angle. So we will use the tan ratio to find x:
\(tan(35)=\frac{12}{x}\) Isolating x:
\(x=\frac{12}{tan(35)}\) so
x = 17.1377 m
Now we have everything we need to find d. We will use 25 degrees as our reference angle, and the side opposite it is 12 and the side adjacent to it is
d + 17.1377, so that is the tan ratio as well:
\(tan(25)=\frac{12}{d+17.1377}\) and simplifying a bit:
\(d+17.1377=\frac{12}{tan(25)}\) and a bit more:
d + 17.1377 = 25.73408 so
d = 8.59, rounded
what shape is both a parallelogram and a rhombus (I ready btw)
Answer:
square
Step-by-step explanation:
a square is a quadrilateral which means it has four sides
Answer:
square
Step-by-step explanation:
What is the domain and range of the function f (x) = a superscript x? a. domain = negative real numbers, range = negative real numbers c. domain = positive real numbers, range = positive real numbers b. domain = all real numbers, range = all real numbers d. domain = real numbers, range = positive real numbers
The domain and range of the function f(x) =a^x, then option (c) Domain = positive real numbers, range = positive real numbers.
The function f(x) = a^x is an exponential function with a base of a, where a is a positive real number. The domain of the function is all real numbers, because we can raise a positive number to any real power.
However, since a is positive, a^x will always be positive, which means that the range of the function is also positive real numbers. Therefore, the correct option is c. Domain = positive real numbers, range = positive real numbers.
Therefore, the correct option is (c) Domain = positive real numbers, range = positive real numbers.
Learn more about function here
brainly.com/question/15352175
#SPJ4
A pilot takes a taxi from the airport to a hotel. The taxi driver charges $2.50 initial charge plus $2.65 per mile. which equation can be used to find y, the total cost of the trip, if x represents the number of miles of the trip.
F. y = 2.50x + 2.65
G. y = 2.65(x + 2.50)
H. y = 2.65x - 2.50
J. y = 2.65x + 2.50
Please help I'm failing Soo bad. If I don't pass I'm in Huge trouble
Answer:
I think its h, I could be wrong.
knock his dreads off
Answer:
oof
Step-by-step explanation:
Answer:?
Step-by-step explanation:
What does this mean
What is the total surface area of the
triangular prism in square centimeters?
6 cm
7 cm
13 cm
A 289 cm
B273 cm
C 357 cm
D315 cm
Answer:
D:315
Step-by-step explanation:
6x7 and then divide by 2 = 21+21 =42 13x7= 91x3= 273+ 42= 315
gmm
2
#x²
13
O
1
2
ABC
3
5x+2
3x + 40
Find the measure of angle 2.
label optional
1
2 3 4
5 6 78 9
XY Z<
7
VI
Xi
+
8
=
V
Answer: \(\angle 2=83^{\circ}\)
Step-by-step explanation:
By the alternate interior angles theorem,
\(5x+2=3x+40\\\\2x=38\\\\x=19\)
This means that \(5x+2=3x+40=97\).
Since \(\angle 2\) and the \(97^{\circ}\) form a linear pair, they add to 180 degrees. So, \(\angle 2=83^{\circ}\)
A dealer can fix 30 dirt bikes in 5 hours. What is the unit rate per hour? How many dirt bike can they
fix in 10 hours?
when conducting a hypothesis test concerning the population mean, and the population standard deviation is known, the value of the test statistic is calculated as
We need to know about test statistic to solve the problem. The formula we will need to calculate test statistic is (sample mean- population mean)/(standard deviation/\(\sqrt{size of sample}\))
Test statistic is a number calculated by a statistical test. It describes how far the observed data is from the null hypothesis of no relationship between variables or no difference among sample groups. The test statistic can be calculated using the sample mean, the population mean and population standard deviation.
test statistic= (sample mean- population mean)/(standard deviation/\(\sqrt{size of sample}\))
Therefore the formula to calculate the test statistic will be (sample mean- population mean)/(standard deviation/\(\sqrt{size of sample}\))
Learn more about test statistic here:
https://brainly.com/question/14287628
#SPJ4
Select the correct answer.
Which value of x makes this equation true?
-12x2(x + 9) = 5(x + 4)
O A.
OB.
O C.
O D.
Answer:
B. -2
Step-by-step explanation:
-12x -2(x+9) = 5(x+4)
*ALWAYS REMEMBER YOUR BODMAS*
-12x -2x - 18 = 5x + 20
*group the like terms*
-14x - 5x = 20 + 18
-19x = 38
x = -2
The value of x that makes the equation true is: x = -2.
B. -2.
Here, we have to find the value of x that makes the equation true, we need to solve for x.
Let's solve the equation step by step:
-12x - 2(x + 9) = 5(x + 4)
Distribute the -2 and 5 on the right side of the equation:
-12x - 2x - 18 = 5x + 20
Combine like terms on both sides:
-14x - 18 = 5x + 20
Move the variable terms (5x) to one side of the equation and the constant terms to the other side.
Let's subtract 5x from both sides:
-14x - 5x - 18 = 5x - 5x + 20
-19x - 18 = 20
Move the constant term (-18) to the other side of the equation.
Let's add 18 to both sides:
-19x - 18 + 18 = 20 + 18
-19x = 38
Finally, divide both sides by -19 to isolate x:
x = 38 / -19
x = -2
So, the value of x that makes the equation true is: x = -2.
B. -2.
To learn more on equation click:
https://brainly.com/question/14468218
#SPJ7
A car leaves Town A for Town B, which are 550 km apart, at an average speed of
72 km/h. At the same time, a truck leaves Town B for Town A and travels along
the same road as the car at an average speed of 38 km/h. Find the time taken for the
two vehicles to meet.
Answer:
b
Step-by-step explanation:
A gas occupies 4.23 l at 2.25 atm. What is the volume at 3.46 atm?
a. 1.84 l
b. 2.75 l
c. 6.50 l
d. 32.9 l
e. 0.364 l
A gas occupies 4.23 l at 2.25 atm. 2.75 L is the volume at 3.46 atm. This can be solved using the concept of Boyle's law.
What is Boyle's law?Boyle's law gives the relationship between pressure and volume of gas.
It states that at constant temperature pressure in inversely proportional to volume.
PV = k
where P - pressure V - volume and k - constant
P₁V₁ = P₂V₂
Given that,
initial pressure (P₁) = 2.25 atm
final pressure (P₂) = 3.46 atm
initial volume (V₁) = 4.23 L
final volume (V₂) = ?
where parameters for the first instance are on the left side and parameters for the second instance are on the right side of the equation
substituting the values in the equation
2.25 atm x 4.23 L = 3.46 atm x V
V = 2.75 L
Thus, the final volume is 2.75 L
To know more about Boyle's law refer to:
https://brainly.com/question/3328692
#SPJ4